Đến nội dung

Hình ảnh

[TOPIC] BẤT ĐẲNG THỨC

bất đẳng thức và cực trị

  • Chủ đề bị khóa Chủ đề bị khóa
Chủ đề này có 72 trả lời

#21
DOTOANNANG

DOTOANNANG

    Đại úy

  • ĐHV Toán Cao cấp
  • 1609 Bài viết

Bài 35:

Bỏ qua $c, a\rightarrow 0,$ chứng minh

$$\forall_{b}\exists_{a\,b\,c}\;a^{2}+ b^{2}+ c^{2}- ab- bc\geq\left | 2ca \right |$$

Giải.

Chọn $a, b, c:= a- b, b- b, c- b,$ có được

$$a^{2}+ b^{2}+ c^{2}- ab- bc- 2ca= \left ( \,c- a\, \right )^{2}\geq 0$$

$$a^{2}+ b^{2}+ c^{2}- ab- bc+ 2ca= \left ( c+ a- 2b \right )^{2}$$


Bài viết đã được chỉnh sửa nội dung bởi DOTOANNANG: 24-06-2021 - 13:16


#22
KietLW9

KietLW9

    Đại úy

  • Điều hành viên THCS
  • 1737 Bài viết

$\boxed{3}$Kỹ thuật sử dụng bất đẳng thức Chebyshev

Bài 36: Chứng minh rằng với $a,b,c$ dương thì: $\frac{a}{b+c}+\frac{b}{c+a}+\frac{c}{a+b}\geqslant \frac{3}{2}$ (Bất đẳng thức Nesbitt)

Bài 37: Cho $a,b,c>0$ và n là số nguyên dương. Chứng minh rằng: $\frac{a^n}{b+c}+\frac{b^n}{c+a}+\frac{c^n}{a+b}\geqslant \frac{(a+b+c)^{n-1}}{2.3^{n-2}}$

Bài 38: Cho $a,b,c>0$ và $\alpha $ là số nguyên dương. Chứng minh rằng: $\frac{a^{\alpha }}{b+c}+\frac{b^{\alpha }}{c+a}+\frac{c^{\alpha }}{a+b}\geqslant \frac{3}{2}.\frac{a^\alpha +b^\alpha +c^\alpha }{a+b+c}$

Bài 39: Cho các số thực $a,b,c>0$ thỏa mãn điều kiện $a^2+b^2+c^2\geqslant1$. Chứng minh rằng: $\frac{a^2}{b+c}+\frac{b^2}{c+a}+\frac{c^2}{a+b}\geqslant \frac{\sqrt{3}}{2}$

Bài 40: Cho $a,b,c,d,e>0$ thỏa mãn $\sum_{cyc}\frac{1}{4+a}=1$. Chứng minh rằng: $\sum_{cyc}\frac{a}{4+a^2}\leqslant 1$

Bài 41: Chứng minh với $a,b,c>0$ thì: $\sum_{cyc}\sqrt{\frac{b+c}{a}}\geqslant 2(\sum_{cyc}\sqrt{\frac{a}{b+c}})$


Bài viết đã được chỉnh sửa nội dung bởi KietLW9: 12-04-2021 - 21:19

Trong cuộc sống không có gì là đẳng thức , tất cả đều là bất đẳng thức  :ukliam2:   :ukliam2: 

 

 

$\text{LOVE}(\text{KT}) S_a (b - c)^2 + S_b (c - a)^2 + S_c (a - b)^2 \geqslant 0\forall S_a,S_b,S_c\geqslant 0$

 

 

 


#23
Hoang72

Hoang72

    Thiếu úy

  • Điều hành viên OLYMPIC
  • 539 Bài viết

Bài 36: Chứng minh rằng với $a,b,c$ dương thì: $\frac{a}{b+c}+\frac{b}{c+a}+\frac{c}{a+b}\geqslant \frac{3}{2}$ (Bất đẳng thức Nesbitt)

Áp dụng bất đẳng thức Chevbyshev cho hai bộ số đơn điệu cùng chiều $(a,b,c);(\frac{1}{b+c},\frac{1}{c+a},\frac{1}{a+b})$ ta có:

$\sum\frac{a}{b+c}\geq \frac{1}{3}(a+b+c)(\sum\frac{1}{a+b})\geq \frac{3}{2}$



#24
Hoang72

Hoang72

    Thiếu úy

  • Điều hành viên OLYMPIC
  • 539 Bài viết

Bài 40: Cho $a,b,c,d,e>0$ thỏa mãn $\sum_{cyc}\frac{1}{4+a}=1$. Chứng minh rằng: $\sum_{cyc}\frac{a}{4+a^2}\leqslant 1$

Ta có $\sum\frac{1}{4+a}=1\Leftrightarrow \sum\frac{a-1}{a+4}=0$.

BĐT tương đương: $\sum \frac{(a-1)(a-4)}{a^2+4}\geq 0\Leftrightarrow \sum(\frac{a-1}{a+4}.\frac{a^2-16}{a^2+4})\geq 0$.

Bất đẳng thức trên đúng theo Chevbyshev do $\frac{a-1}{a+4}\geq \frac{b-1}{b+4}\Leftrightarrow \frac{a^2-16}{a^4+4}\geq\frac{b^2-16}{b^2+4}$.



#25
KietLW9

KietLW9

    Đại úy

  • Điều hành viên THCS
  • 1737 Bài viết

$\boxed{4}$Kỹ thuật dồn biến (Một bài dồn biến theo mình là cực hay)

Bài 42: Cho $a,b,c$ không âm. Chứng minh rằng: $a^3+b^3+c^3+9abc+4(a+b+c)\geqslant 8(ab+bc+ca)$


Bài viết đã được chỉnh sửa nội dung bởi KietLW9: 14-04-2021 - 12:22

Trong cuộc sống không có gì là đẳng thức , tất cả đều là bất đẳng thức  :ukliam2:   :ukliam2: 

 

 

$\text{LOVE}(\text{KT}) S_a (b - c)^2 + S_b (c - a)^2 + S_c (a - b)^2 \geqslant 0\forall S_a,S_b,S_c\geqslant 0$

 

 

 


#26
KietLW9

KietLW9

    Đại úy

  • Điều hành viên THCS
  • 1737 Bài viết

$\boxed{5}$Phần tiếp theo là bất đẳng thức lớp 8 được mình chọn lọc trong các đề thi tỉnh, huyện. 

Bài 42: Chứng minh rằng: $a(a+b)(a^2+b^2)(a^4+b^4)+b(b+c)(b^2+c^2)(b^4+c^4)+c(c+a)(c^2+a^2)(c^4+a^4)\geqslant 0$

Bài 43: Cho $a,b,c,d,e$ không âm thỏa mãn $a+b+c+d+e=1$. Chứng minh rằng: $ab+bc+cd+de\leqslant \frac{1}{4}$

Bài 44: Cho $a,b,c$ là độ dài các cạnh của một tam giác có chu vi 1. Chứng minh rằng: $\frac{1-2a^2}{(1-2a)^2}+\frac{1-2b^2}{(1-2b)^2}+\frac{1-2c^2}{(1-2c)^2}\geqslant 21$

Bài 45: Cho $a,b,c$ thỏa mãn $a^2+b^2+c^2=2$. Tìm GTNN, GTLN của biểu thức: $M=a+b+c-abc$

Bài 46: Cho $a,b,c,d>0$ thỏa mãn $a+b+c+d\geqslant 2016$. Tìm GTNN của biểu thức: $M=\frac{a^4}{(a^2+b^2)(a+b)}+\frac{b^4}{(b^2+c^2)(b+c)}+\frac{c^4}{(c^2+d^2)(c+d)}+\frac{d^4}{(d^2+a^2)(d+a)}$

Bài 47: Cho $x,y$ là hai số dương thỏa mãn $x+y=1$. Chứng minh rằng: $3(3x-2)^2+\frac{8x}{y}\geqslant 7$

Bài 48: Cho $a,b,c$ thỏa mãn $1\leqslant a\leqslant b\leqslant c\leqslant 2$. Chứng minh rằng: $(a+b+c)(\frac{1}{a}+\frac{1}{b}+\frac{1}{c})\leqslant 10$


Bài viết đã được chỉnh sửa nội dung bởi KietLW9: 17-04-2021 - 19:36

Trong cuộc sống không có gì là đẳng thức , tất cả đều là bất đẳng thức  :ukliam2:   :ukliam2: 

 

 

$\text{LOVE}(\text{KT}) S_a (b - c)^2 + S_b (c - a)^2 + S_c (a - b)^2 \geqslant 0\forall S_a,S_b,S_c\geqslant 0$

 

 

 


#27
Hoang72

Hoang72

    Thiếu úy

  • Điều hành viên OLYMPIC
  • 539 Bài viết

Bài 42: Chứng minh rằng: $a(a+b)(a^2+b^2)(a^4+b^4)+b(b+c)(b^2+c^2)(b^4+c^4)+c(c+a)(c^2+a^2)(c^4+a^4)\geqslant 0$

Đặt A = $\sum_{cyc}a(a+b)(a^2+b^2)(a^4+b^4)$. B = $\sum_{cyc}b(a+b)(a^2+b^2)(a^4+b^4)$.

Dễ thấy A - B = 0 nên A = B.

Mặt khác do A + B = $\sum(a+b)^2(a^2+b^2)(a^4+b^4)\geq 0$ nên $A\geq 0$.

Dấu "=" xảy ra khi a = b = c = 0.



#28
KietLW9

KietLW9

    Đại úy

  • Điều hành viên THCS
  • 1737 Bài viết

Thêm một bài mà mình nghĩ rất tư duy đối với học sinh lớp 8 :D

Bài 49: Cho $a,b,c$ không âm thỏa mãn $a+b+c=1$. Chứng minh rằng: $8(a^2+b^2+c^2)(a^2b^2+b^2c^2+c^2a^2+abc)\leqslant ab+bc+ca$


Trong cuộc sống không có gì là đẳng thức , tất cả đều là bất đẳng thức  :ukliam2:   :ukliam2: 

 

 

$\text{LOVE}(\text{KT}) S_a (b - c)^2 + S_b (c - a)^2 + S_c (a - b)^2 \geqslant 0\forall S_a,S_b,S_c\geqslant 0$

 

 

 


#29
KietLW9

KietLW9

    Đại úy

  • Điều hành viên THCS
  • 1737 Bài viết

Đặt A = $\sum_{cyc}a(a+b)(a^2+b^2)(a^4+b^4)$. B = $\sum_{cyc}b(a+b)(a^2+b^2)(a^4+b^4)$.

Dễ thấy A - B = 0 nên A = B.

Mặt khác do A + B = $\sum(a+b)^2(a^2+b^2)(a^4+b^4)\geq 0$ nên $A\geq 0$.

Dấu "=" xảy ra khi a = b = c = 0.

Ta có: $a(a+b)=a^2+ab=(\frac{a^2}{2}+\frac{2ab}{2}+\frac{b^2}{2})+(\frac{a^2}{2}-\frac{b^2}{2})=\frac{(a+b)^2}{2}+\frac{a^2-b^2}{2}\geqslant \frac{a^2-b^2}{2}$

$\Rightarrow a(a+b)(a^2+b^2)(a^4+b^4)\geqslant \frac{(a^2-b^2)(a^2+b^2)(a^4+b^4)}{2}=\frac{a^8-b^8}{2}$

Tương tự: $b(b+c)(b^2+c^2)(b^4+c^4)\geqslant \frac{b^8-c^8}{2}$; $c(c+a)(c^2+a^2)(c^4+a^4)\geqslant \frac{c^8-a^8}{2}$

Cộng theo vế ba bất đẳng thức trên, ta được: $a(a+b)(a^2+b^2)(a^4+b^4)+b(b+c)(b^2+c^2)(b^4+c^4)+c(c+a)(c^2+a^2)(c^4+a^4)\geqslant \frac{a^8-b^8+b^8-c^8+c^8-a^8}{2}=0(Q.E.D)$

Đẳng thức xảy ra khi $a = b = c = 0$  :D


Trong cuộc sống không có gì là đẳng thức , tất cả đều là bất đẳng thức  :ukliam2:   :ukliam2: 

 

 

$\text{LOVE}(\text{KT}) S_a (b - c)^2 + S_b (c - a)^2 + S_c (a - b)^2 \geqslant 0\forall S_a,S_b,S_c\geqslant 0$

 

 

 


#30
KietLW9

KietLW9

    Đại úy

  • Điều hành viên THCS
  • 1737 Bài viết

Bài 45: Cho $a,b,c$ thỏa mãn $a^2+b^2+c^2=2$. Tìm GTNN, GTLN của biểu thức: $M=a+b+c-abc$

Ta luôn có: $2bc\leqslant b^2+c^2\leqslant a^2+b^2+c^2=2\Rightarrow bc\leqslant 1\Rightarrow bc-1\leqslant 0$

Áp dụng bất đẳng thức Cauchy-Schwarz: $M^2=(a+b+c-abc)^2=(a(1-bc)+(b+c).1)^2\leqslant [a^2+(b+c)^2][(1-bc)^2+1^2]=(2+2bc)(b^2c^2-2bc+2)=2b^2c^2(bc-1)+4\leqslant 4\Rightarrow -2\leqslant M\leqslant 2$

Vậy:

$\circ$ GTLN của M là 2 đạt được khi $a = 0; b = c = 1$ hoặc các hoán vị

$\circ$ GTNN của M là -2 đạt được khi $a = 0; b = c = -1$ hoặc các hoán vị


Bài viết đã được chỉnh sửa nội dung bởi KietLW9: 24-12-2021 - 20:46

Trong cuộc sống không có gì là đẳng thức , tất cả đều là bất đẳng thức  :ukliam2:   :ukliam2: 

 

 

$\text{LOVE}(\text{KT}) S_a (b - c)^2 + S_b (c - a)^2 + S_c (a - b)^2 \geqslant 0\forall S_a,S_b,S_c\geqslant 0$

 

 

 


#31
KietLW9

KietLW9

    Đại úy

  • Điều hành viên THCS
  • 1737 Bài viết

Thêm một bài mà mình nghĩ rất tư duy đối với học sinh lớp 8 :D

Bài 49: Cho $a,b,c$ không âm thỏa mãn $a+b+c=1$. Chứng minh rằng: $8(a^2+b^2+c^2)(a^2b^2+b^2c^2+c^2a^2+abc)\leqslant ab+bc+ca$

Đặt $ab+bc+ca=x$ thì $a^2b^2+b^2c^2+c^2a^2+abc\leqslant a^2b^2+b^2c^2+c^2a^2+2abc(a+b+c)=(ab+bc+ca)^2=x^2$ và $a^2+b^2+c^2=(a+b+c)^2-2(ab+bc+ca)=1-2x$

Như vậy, ta cần chứng minh: $8(1-2x)x^2\leqslant x\Leftrightarrow x(4x-1)^2\geqslant 0(True)$

Đẳng thức xảy ra khi trong 3 số $a,b,c$ có 2 số bằng $\frac{1}{2}$ và 1 số bằng 0


Trong cuộc sống không có gì là đẳng thức , tất cả đều là bất đẳng thức  :ukliam2:   :ukliam2: 

 

 

$\text{LOVE}(\text{KT}) S_a (b - c)^2 + S_b (c - a)^2 + S_c (a - b)^2 \geqslant 0\forall S_a,S_b,S_c\geqslant 0$

 

 

 


#32
KietLW9

KietLW9

    Đại úy

  • Điều hành viên THCS
  • 1737 Bài viết

Bài 43: Cho $a,b,c,d,e$ không âm thỏa mãn $a+b+c+d+e=1$. Chứng minh rằng: $ab+bc+cd+de\leqslant \frac{1}{4}$

Vì $a,b,c,d,e$ không âm nên $ab+bc+cd+de\leqslant ab+bc+cd+de+be+ad=(b+d)(a+c+e)\leqslant \frac{(a+b+c+d+e)^2}{4}=\frac{1}{4}$

 

Bài 47: Cho $x,y$ là hai số dương thỏa mãn $x+y=1$. Chứng minh rằng: $3(3x-2)^2+\frac{8x}{y}\geqslant 7$

Vì $x+y=1$ nên ta cần chứng minh: $3(3x-2)^2+\frac{8x}{1-x}\geqslant 7\Leftrightarrow (5-3x)(3x-1)^2\geqslant 0(True)$

Đẳng thức xảy ra khi $x=\frac{1}{3};y=\frac{2}{3}$


Bài viết đã được chỉnh sửa nội dung bởi KietLW9: 17-04-2021 - 19:42

Trong cuộc sống không có gì là đẳng thức , tất cả đều là bất đẳng thức  :ukliam2:   :ukliam2: 

 

 

$\text{LOVE}(\text{KT}) S_a (b - c)^2 + S_b (c - a)^2 + S_c (a - b)^2 \geqslant 0\forall S_a,S_b,S_c\geqslant 0$

 

 

 


#33
KietLW9

KietLW9

    Đại úy

  • Điều hành viên THCS
  • 1737 Bài viết

Bài 44: Cho $a,b,c$ là độ dài các cạnh của một tam giác có chu vi 1. Chứng minh rằng: $\frac{1-2a^2}{(1-2a)^2}+\frac{1-2b^2}{(1-2b)^2}+\frac{1-2c^2}{(1-2c)^2}\geqslant 21$

Ta có: $\sum_{cyc}\frac{1-2a^2}{(1-2a)^2}\geqslant \sum_{cyc}\frac{-20a^2+12a-1}{(1-2a)^2}=\sum_{cyc}\frac{(10a-1)(1-2a)}{(1-2a)^2}=\sum_{cyc}\frac{10a-1}{1-2a}=\sum_{cyc}\frac{-5(1-2a)+4}{1-2a}=-15+\sum_{cyc}\frac{4}{1-2a}=-15+\sum_{cyc}\frac{4}{b+c-a}\geqslant -15+\frac{36}{a+b+c}=21$

Đẳng thức xảy ra khi $a=b=c=\frac{1}{3}$


Trong cuộc sống không có gì là đẳng thức , tất cả đều là bất đẳng thức  :ukliam2:   :ukliam2: 

 

 

$\text{LOVE}(\text{KT}) S_a (b - c)^2 + S_b (c - a)^2 + S_c (a - b)^2 \geqslant 0\forall S_a,S_b,S_c\geqslant 0$

 

 

 


#34
CloudSup

CloudSup

    Binh nhì

  • Thành viên mới
  • 11 Bài viết

Câu 50: Cho a,b,c là các số thực thỏa mãn $ab+bc+ca\geq 11$. CMR:

$(a^2+1)(b^2+1)(c^2+1)\geq 100$



#35
PDF

PDF

    Trung sĩ

  • Thành viên
  • 197 Bài viết

$\boxed{4}$Kỹ thuật dồn biến (Một bài dồn biến theo mình là cực hay)

Bài 42: Cho $a,b,c$ không âm. Chứng minh rằng: $a^3+b^3+c^3+9abc+4(a+b+c)\geqslant 8(ab+bc+ca)$

Cách dồn biến:

Đặt $$f(a,b,c)=a^{3}+b^{3}+c^{3}+9abc+4(a+b+c)-8(bc+ca+ab).$$

Ta có: $$\left[f(a,b,c)-f(a,\frac{b+c}{2},\frac{b+c}{2})\right][f(a,b,c)-f(a,b+c,0)]=-\frac{bc(b-c)^{2}(9a-3b-3c-8)^{2}}{4}\leq 0.$$

Do đó một trong hai thừa số phải không âm, tức là ta cần kiểm tra BĐT trong trường hợp có hai biến bằng nhau hoặc có một biến bằng 0.

Trường hợp 1: Hai biến bằng nhau.

Ta có $$f(a,t,t)=a^{3}+(9t^{2}-16t+4)a+2t(2-t)^{2}.$$

Rõ ràng ta chỉ cần xét trường hợp $9t^{2}-16t+4<0$, tức là $$\frac{8-2\sqrt{7}}{9}\leq t\leq \frac{8+2\sqrt{7}}{9}<2.$$

Áp dụng BĐT AM-GM, ta chỉ cần chứng minh

$$27t^{2}(2-t)^{4}+(9t^{2}-16t+4)^{3}\geq 0,$$

hay $$4(189t^{4}-648t^{3}+648t^{2}-160t+16)(t-1)^{2}\geq 0.$$

Với $t\leq 1$, BĐT là hiển nhiên. Xét $t>1$.

Áp dụng BĐT AM-GM có:

$$VT\geq 108\sqrt{42}t^{3}-160t+16>108\sqrt{42}t^{2}-160t+16\geq (48\sqrt[4]{378}-160)t>0.$$

Trường hợp 2: Có một biến bằng 0.

Ta có $$f(a,t,0)=a^{3}-4(2t-1)a+t(t^{2}+4).$$

Ta cũng dùng BĐT AM-GM để khử $a$ rồi chứng minh BĐT một biến giống trường hợp trên.

Đẳng thức xảy ra khi $a=b=c=1$ hoặc $a=0,b=c=2$ và các hoán vị tương ứng. $\square$

PS: Có cách khác không sử dụng dồn biến rất gọn và đẹp. :)


Bài viết đã được chỉnh sửa nội dung bởi PDF: 18-04-2021 - 00:10


#36
KietLW9

KietLW9

    Đại úy

  • Điều hành viên THCS
  • 1737 Bài viết

Cách dồn biến:

Đặt $$f(a,b,c)=a^{3}+b^{3}+c^{3}+9abc+4(a+b+c)-8(bc+ca+ab).$$

Ta có: $$\left[f(a,b,c)-f(a,\frac{b+c}{2},\frac{b+c}{2})\right][f(a,b,c)-f(a,b+c,0)]=-\frac{bc(b-c)^{2}(9a-3b-3c-8)^{2}}{4}\leq 0.$$

Do đó một trong hai thừa số phải không âm, tức là ta cần kiểm tra BĐT trong trường hợp có hai biến bằng nhau hoặc có một biến bằng 0.

Trường hợp 1: Hai biến bằng nhau.

Ta có $$f(a,t,t)=a^{3}+(9t^{2}-16t+4)a+2t(2-t)^{2}.$$

Rõ ràng ta chỉ cần xét trường hợp $9t^{2}-16t+4<0$, tức là $$\frac{8-2\sqrt{7}}{9}\leq t\leq \frac{8+2\sqrt{7}}{9}<2.$$

Áp dụng BĐT AM-GM, ta chỉ cần chứng minh

$$27t^{2}(2-t)^{4}+(9t^{2}-16t+4)^{3}\geq 0,$$

hay $$4(189t^{4}-648t^{3}+648t^{2}-160t+16)(t-1)^{2}\geq 0.$$

Với $t\leq 1$, BĐT là hiển nhiên. Xét $t>1$.

Áp dụng BĐT AM-GM có:

$$VT\geq 108\sqrt{42}t^{3}-160t+16>108\sqrt{42}t^{2}-160t+16\geq (48\sqrt[4]{378}-160)t>0.$$

Trường hợp 2: Có một biến bằng 0.

Ta có $$f(a,t,0)=a^{3}-4(2t-1)a+t(t^{2}+4).$$

Ta cũng dùng BĐT AM-GM để khử $a$ rồi chứng minh BĐT một biến giống trường hợp trên.

Đẳng thức xảy ra khi $a=b=c=1$ hoặc $a=0,b=c=2$ và các hoán vị tương ứng. $\square$

PS: Có cách khác không sử dụng dồn biến rất gọn và đẹp. :)

Mình nghĩ là $p,q,r$


Trong cuộc sống không có gì là đẳng thức , tất cả đều là bất đẳng thức  :ukliam2:   :ukliam2: 

 

 

$\text{LOVE}(\text{KT}) S_a (b - c)^2 + S_b (c - a)^2 + S_c (a - b)^2 \geqslant 0\forall S_a,S_b,S_c\geqslant 0$

 

 

 


#37
truonganh2812

truonganh2812

    Lính mới

  • Thành viên mới
  • 5 Bài viết

$\boxed{4}$Kỹ thuật dồn biến (Một bài dồn biến theo mình là cực hay)

Bài 42: Cho $a,b,c$ không âm. Chứng minh rằng: $a^3+b^3+c^3+9abc+4(a+b+c)\geqslant 8(ab+bc+ca)$

$a^3+b^3+c^3+9abc+4(a+b+c)\geq 4\sqrt{(\sum a^3+9abc)(a+b+c)} (\sum a^3+9abc)(a+b+c)\geq 4(\sum ab)^2. PQR...p^4+4q^2-5p^2q+6rp\geq 0. p^2q+3rp\geq 4q^2 a^3b+b^3a\geq 2(ab)^2$



#38
KietLW9

KietLW9

    Đại úy

  • Điều hành viên THCS
  • 1737 Bài viết

$a^3+b^3+c^3+9abc+4(a+b+c)\geq 4\sqrt{(\sum a^3+9abc)(a+b+c)} (\sum a^3+9abc)(a+b+c)\geq 4(\sum ab)^2. PQR...p^4+4q^2-5p^2q+6rp\geq 0. p^2q+3rp\geq 4q^2 a^3b+b^3a\geq 2(ab)^2$

Không biết ý anh có phải như thế này không:

Áp dụng bất đẳng thức Cauchy, ta được: $a^3+b^3+c^3+9abc+4(a+b+c)\geqslant 2\sqrt{(a^3+b^3+c^3+9abc).4(a+b+c)}=4\sqrt{(a^3+b^3+c^3+9abc)(a+b+c)}$

Đến đây, ta cần chứng minh: $(a^3+b^3+c^3+9abc)(a+b+c)\geqslant 4(ab+bc+ca)^2$

Đặt $a+b+c=p;ab+bc+ca=q;abc=r$ thì ta có: $(a^3+b^3+c^3+9abc)(a+b+c)=(a^4+b^4+c^4)+[ab(a^2+b^2)+bc(b^2+c^2)+ca(c^2+a^2)]+9abc(a+b+c)=(p^4-4p^2q+2q^2+4pr)+(p^2q-2q^2-pr)+9pr=p^4-3p^2q+12pr$

Theo bất đẳng thức Schur bậc 3, ta có: $p^3+9r\geqslant 4pq\Rightarrow p^4+9pr\geqslant 4p^2q\Leftrightarrow p^4-3p^2q+12pr\geqslant p^2q+3pr$

Như vậy, ta quy về chứng minh: $p^2q+3pr\geqslant 4q^2$

Thật vậy, ta có: $p^2q+3pr=(a+b+c)^2(ab+bc+ca)+3abc(a+b+c)=(a^3b+ab^3)+(b^3c+bc^3)+(c^3a+ca^3)+2(a^2b^2+b^2c^2+c^2a^2)+8abc(a+b+c)\geqslant2(a^2b^2+b^2c^2+c^2a^2)+2(a^2b^2+b^2c^2+c^2a^2)+8abc(a+b+c)=4(ab+bc+ca)^2(Q.E.D)$ 


Trong cuộc sống không có gì là đẳng thức , tất cả đều là bất đẳng thức  :ukliam2:   :ukliam2: 

 

 

$\text{LOVE}(\text{KT}) S_a (b - c)^2 + S_b (c - a)^2 + S_c (a - b)^2 \geqslant 0\forall S_a,S_b,S_c\geqslant 0$

 

 

 


#39
KietLW9

KietLW9

    Đại úy

  • Điều hành viên THCS
  • 1737 Bài viết

Câu 50: Cho a,b,c là các số thực thỏa mãn $ab+bc+ca\geq 11$. CMR:

$(a^2+1)(b^2+1)(c^2+1)\geq 100$

Không biết có trùng hợp hay không nhưng bài này cũng đồng thời xuất hiện trong bài $\boxed{15}$ ở MARATHON Bất đẳng thức của anh Long

Mình xin nêu tiếp cho Topic bài 51 do mình tự sáng tác (tất nhiên là mình sẽ giải sau 2 ngày, mong là không có gì sai sót :D )

$\boxed{51}$Cho $a,b,c,d$ là các số dương thỏa mãn $abcd\leqslant 1$. Chứng minh rằng: $(a+b+c+d)(\frac{1}{a}+\frac{1}{b}+\frac{1}{c}+\frac{1}{d})\geqslant 3a\sqrt[3]{a}+3b\sqrt[3]{b}+3c\sqrt[3]{c}+3d\sqrt[3]{d}+4$


Trong cuộc sống không có gì là đẳng thức , tất cả đều là bất đẳng thức  :ukliam2:   :ukliam2: 

 

 

$\text{LOVE}(\text{KT}) S_a (b - c)^2 + S_b (c - a)^2 + S_c (a - b)^2 \geqslant 0\forall S_a,S_b,S_c\geqslant 0$

 

 

 


#40
PDF

PDF

    Trung sĩ

  • Thành viên
  • 197 Bài viết

Không biết ý anh có phải như thế này không:

Áp dụng bất đẳng thức Cauchy, ta được: $a^3+b^3+c^3+9abc+4(a+b+c)\geqslant 2\sqrt{(a^3+b^3+c^3+9abc).4(a+b+c)}=4\sqrt{(a^3+b^3+c^3+9abc)(a+b+c)}$

Đến đây, ta cần chứng minh: $(a^3+b^3+c^3+9abc)(a+b+c)\geqslant 4(ab+bc+ca)^2$

Đặt $a+b+c=p;ab+bc+ca=q;abc=r$ thì ta có: $(a^3+b^3+c^3+9abc)(a+b+c)=(a^4+b^4+c^4)+[ab(a^2+b^2)+bc(b^2+c^2)+ca(c^2+a^2)]+9abc(a+b+c)=(p^4-4p^2q+2q^2+4pr)+(p^2q-2q^2-pr)+9pr=p^4-3p^2q+12pr$

Theo bất đẳng thức Schur bậc 3, ta có: $p^3+9r\geqslant 4pq\Rightarrow p^4+9pr\geqslant 4p^2q\Leftrightarrow p^4-3p^2q+12pr\geqslant p^2q+3pr$

Như vậy, ta quy về chứng minh: $p^2q+3pr\geqslant 4q^2$

Thật vậy, ta có: $p^2q+3pr=(a+b+c)^2(ab+bc+ca)+3abc(a+b+c)=(a^3b+ab^3)+(b^3c+bc^3)+(c^3a+ca^3)+2(a^2b^2+b^2c^2+c^2a^2)+8abc(a+b+c)\geqslant2(a^2b^2+b^2c^2+c^2a^2)+2(a^2b^2+b^2c^2+c^2a^2)+8abc(a+b+c)=4(ab+bc+ca)^2(Q.E.D)$ 

Có thể xử lý gọn hơn nữa như thế này:

BĐT tương đương với $$a^{4}+b^{4}+c^{4}+abc(a+b+c)+bc(b^{2}+c^{2})+ca(c^{2}+a^{2})+ab(a^{2}+b^{2})\geq 4(b^{2}c^{2}+c^{2}a^{2}+a^{2}b^{2}).$$

Áp dụng BĐT Schur bậc ba:

$$abc\geq (b+c-a)(c+a-b)(a+b-c)$$

$$\Rightarrow abc(a+b+c)\geq (a+b+c)(b+c-a)(c+a-b)(a+b-c)=2(b^{2}c^{2}+c^{2}a^{2}+a^{2}b^{2})-a^{4}-b^{4}-c^{4}$$

$$\Leftrightarrow a^{4}+b^{4}+c^{4}+abc(a+b+c)\geq 2(b^{2}c^{2}+c^{2}a^{2}+a^{2}b^{2}).$$

Lại theo BĐT AM-GM: $$bc(b^{2}+c^{2})+ca(c^{2}+a^{2})+ab(a^{2}+b^{2})\geq 2(b^{2}c^{2}+c^{2}a^{2}+a^{2}b^{2}).$$

Cộng lại có ngay đpcm. $\square$







Được gắn nhãn với một hoặc nhiều trong số những từ khóa sau: bất đẳng thức và cực trị

2 người đang xem chủ đề

0 thành viên, 2 khách, 0 thành viên ẩn danh